Sự khác biệt giữa đầu và đuôi


12

Hãy xem xét một chuỗi lần lật của một đồng xu không thiên vị. Hãy biểu thị giá trị tuyệt đối của số chênh lệch giữa số lượng người đứng đầu trên đuôi nhìn thấy trong lần đầu tiên flips. Xác định . rằng và .nHiiH=maxiHiE[Hi]=Θ(i)E[H]=Θ(n)

Vấn đề này xuất hiện trong chương đầu tiên của 'Thuật toán ngẫu nhiên' của Raghavan và Motwani, vì vậy có lẽ có một bằng chứng cơ bản về tuyên bố trên. Tôi không thể giải quyết nó, vì vậy tôi sẽ đánh giá cao bất kỳ sự giúp đỡ nào.

Câu trả lời:


9

Tiền xu của bạn lật tạo thành một bước đi ngẫu nhiên một chiều bắt đầu từ , với , mỗi tùy chọn có xác suất . Bây giờvà vì vậy . Thật dễ dàng để tính (đây chỉ là phương sai), và do đó từ độ lồi. Chúng tôi cũng biết rằng được phân phối gần như bình thường với giá trị trung bình bằng 0 và phương sai , và do đó bạn có thể tính .X 0 = 0 X i + 1 = X i ± 1 1 / 2 H i = | X i | H 2 i = X 2 i E [ X 2 i ] = i E [ H i ] X0,X1,X0=0Xi+1=Xi±11/2Hi=|Xi|Hi2=Xi2E[Xi2]=i XtôiiE[Hi]E[Hi]E[Hi2]=iXiiE[Hi](2/π)i

Đối với , chúng ta có luật logarit lặp , điều này (có lẽ) khiến chúng ta mong đợi một cái gì đó lớn hơn một chút so với . Nếu bạn tốt với giới hạn trên của , bạn có thể sử dụng độ lệch lớn bị ràng buộc cho mỗi và sau đó liên kết bị ràng buộc, mặc dù điều đó bỏ qua thực tế là có liên quan.E[maxinHi]˜ O (nXiXiO~(n)XiXi

Chỉnh sửa: Khi nó xảy ra, do nguyên tắc phản ánh, hãy xem câu hỏi này . Vì vậy, kể từ . Bây giờ và do đóE [ max i n X i ]Pr[maxinXi=k]=Pr[Xn=k]+Pr[Xn=k+1] Pr[Hn=k]=Pr[Xn=k]+Pr[Xn=-k]=2Pr[Xn=k] max i n X i + max i n (- X i

E[maxinXi]=k0k(Pr[Xn=k]+Pr[Xn=k+1])=k1(2k1)Pr[Xn=k]=k12kPr[Xn=k]12+12Pr[Xn=0]=E[Hn]+Θ(1),
Pr[Hn=k]=Pr[Xn=k]+Pr[Xn=k]=2Pr[Xn=k]E[maxinHi]2E[Hn]+Θ(1)=O(
maxinXi+maxin(Xi)2maxinHimaxinXi+maxin(Xi),
E[maxinHi]2E[Hn]+Θ(1)=O(n). Các hướng khác là tương tự.

Sau khi chúng tôi chứng minh , chúng tôi không thể nói rằng với chúng tôi có kết quả thứ hai, tức là không có lớn hơn hơn . i=nE[H i ]Θ(E[Hi]=Θ(i)i=nE[Hi]Θ(n)
chazisop

1
Nếu là độc lập, thì kết luận sẽ không đúng, vì bạn thực sự mong đợi một số giá trị này sẽ lớn hơn một chút so với kỳ vọng. Nói chung không đúng khi . E [ max ( A , B ) ] = max ( E [ A ] , E [ B ] )HiE[max(A,B)]=max(E[A],E[B])
Yuval Filmus

1
Luật logarit lặp không áp dụng ở đây, vì là cố định và chúng tôi không bình thường hóa bởi . Câu trả lời cho là . i E max i n H i θ ( niEmaxinHiθ(n)
Peter Shor

+1 cho phần đầu tiên. nhưng tôi thực sự không hiểu phần thứ hai (bạn có thể giải thích thêm plz). Điều này không có nghĩa là nó không đúng.
AJed

1
Bằng chứng tốt đẹp. Nhưng tôi bế tắc về cách chứng minh là giới hạn dưới của ? Có vẻ như câu trả lời đề cập không có chi tiết về giới hạn dưới. E(Hi)nE(Hi)
konjac

2

Bạn có thể sử dụng phân phối nửa bình thường để chứng minh câu trả lời.

Phân phối nửa bình thường nói rằng nếu là phân phối chuẩn với trung bình 0 và phương sai , thìtheo phân phối một nửa với trung bình và phương sai . Điều này đưa ra câu trả lời bắt buộc, vì phương sai của bước đi bình thường là và bạn có thể ước chừng phân phối thành phân phối chuẩn bằng định lý giới hạn trung tâm.σ 2 | X |Xσ2|X|σ2πσ2(12/π)σ2nX

X là tổng của bước đi ngẫu nhiên như Yuval Filmus đã đề cập.


Tôi không thích điều này mà tôi đã đăng .. mặc dù nó cho giới hạn dưới, không có gì có thể được nói về giới hạn trên. Tôi đã cố gắng sử dụng một đối số phân phối tối đa để giải quyết nó, hóa ra đó là một sự tích hợp xấu xí. Nhưng nó là tốt để biết tất cả các phân phối.
AJed

2

Trong đầu tiên , giả sử chúng ta có k đuôi, sau đó H 2 i = 2 | tôi - k | . Do đó, E ( H 2 i )2ikH2i=2|ik| Sử dụngxấp xỉ Stirling của, chúng ta biếtE(H2i)=Θ(

E(H2i)=2k=0i(2ik)(12)2i2(ik)=(12)2i2[ik=0i(2ik)k=0ik(2ik)]=(12)2i2[i(22i+(2ii))/22ik=0i1(2i1k)]=(12)2i2i[22i1+(2ii)/2222i1/2]=2i(2ii)/22i.
.E(H2i)=Θ(2i)

i<k2i
Khi sử dụng trang web của chúng tôi, bạn xác nhận rằng bạn đã đọc và hiểu Chính sách cookieChính sách bảo mật của chúng tôi.
Licensed under cc by-sa 3.0 with attribution required.